2
$\begingroup$

Suppose that we are given some finitely generated group $ G $ and some finite index subgroup of it $ H $. Given a finite generating symmetric generating set $ S \subset G $, we can define the word metric on $ G $:

$$ d_{S}(g,x):= |g^{-1}x|=min(n\in \mathbb{N} : \exists s_{1},...,s_{n}\in S, g^{-1}h=s_{1}\cdot s_{2}\cdot \dots\cdot s_{n}=g^{-1}x) $$ for all $ g,x \in G $

Similarly, take some symmetric generating set $ T \subset H $ and consider the word metric $ d_{T}^{\prime} $ on $H$. It is well known that these are quasi-isometric on H - for some constants $ K>0 , C>0$, we have:

$$ \frac{1}{K} d_S(h,y)-C \le d^{\prime}_{T}(h,y) \le K d_S(h,y)+C \;\;\forall h,y\in H $$

This property is used many times in geometric problems, and it is quite useful.

However, i'm studying the horofunction boundary (especially for nilpotent groups), in which expressions of the form $ d_S(g,x)-d_S(g,x_{0}) $ appear all the time. I want to know how to relate these to expressions of the form $ d_{T}^{\prime}(g,x)-d_{T}^{\prime}(g,x_{0}) $.

This motivates the following questions:

  1. Given a symmetric generating set $ S \subset G $, can one always find a symmetric generating set $T \subset H$ and some $C>0$ such that $$ d_{T}^{\prime}(y,h)-C \le d_{S}(y,h) \le d_{T}^{\prime}(y,h)+C \;\; \forall y,h\in H $$ is satisfied? (Edit: Solved as negative by Michael Stoll - Take $G=\mathbb{Z}$ with $S=\{\pm 1\}$ and $H=2\mathbb{Z}$).

  2. Can one find some symmetric generating sets $ S \subset G $ and $ T \subset H $ and some $C>0$ such that $$ d_{T}^{\prime}(y,h)-C \le d_{S}(y,h) \le d_{T}^{\prime}(y,h)+C \;\; \forall y,h\in H $$ is satisfied?

  3. Does the answer to one of the prior questions change if $H$ is assumed to be nilpotent? (Edit: part 1 is solved by Michael Stoll with the same example above)

Edit: As a response to Michael's example, I would like to raise another question:

  1. Does the answer to one of the prior questions change if we relax our demand, replacing the wanted inequality by $$ Kd_{T}^{\prime}(y,h)-C \le d_{S}(y,h) \le Kd_{T}^{\prime}(y,h)+C \;\; \forall y,h\in H $$ for some constants $C,K>0$?
$\endgroup$
10
  • 4
    $\begingroup$ The answer to Question 1 is "No". Take $G = \mathbb Z$ and $S = \{\pm 1\}$, $H = 2{\mathbb Z}$. For any $h \in H$, $|h|_S = |h|$, but $|h|_T$ will grow like $|h|/\max\{|t| : t \in T\} \le |h|/2$. $\endgroup$ Nov 16, 2014 at 15:42
  • 1
    $\begingroup$ Here is an example of a positive answer to Question 4. Take $G = \langle x, y \rangle$, the free group on two generators, with standard generating set $S = \{x,x^{-1},y,y^{-1}\}$. Take for $H$ the "even" subgroup (whose elements are represented by words of even length). Then with $T$ the set of all elements of length 2, the inequality holds with $K = 2$, $C = 0$ (i.e., it is actually an equality). The same construction will work when all relations among elements of $S$ have even length and $H$ is the subgroup of elements of even length. $\endgroup$ Nov 16, 2014 at 16:50
  • 1
    $\begingroup$ Here is another situation in which even 2 is true as an equality. Suppose that $H$ is a normal subgroup and that there is a finite subgroup $K$ such that $G = HK$. Let $T$ be any symmetric generating set of $H$ that is closed under conjugation by elements from $K$ and contains $H \cap K \setminus \{1\}$. Let $S = T \cup K \setminus \{1\}$. Since $T \subset S$, we trivially have $|h|_T \ge |h|_S$. On the other hand, any $S$-word $t_0 k_1 t_1 k_2 \cdots k_n t_n$ (with $T$-words $t_j$ and $k_j \in K$) can be rewritten as an $S$-word $t'_0 t'_1 \cdots t'_n k_1 \cdots k_n$ of the same length. ... $\endgroup$ Nov 18, 2014 at 19:10
  • 1
    $\begingroup$ ... If the result is in $H$, then $k_1 k_2 \cdots k_n =: k \in H$, so the element can be written as a $T$-word of at most the same length (either $k = 1$, then the $K$-part can be left out, or else $n$ must be $\ge 1$, and the $K$-part can be replaced by one generator $k \in K \cap H \setminus \{1\} \subset T$). This shows $|h|_T \le |h|_S$ for $h \in H$. $\endgroup$ Nov 18, 2014 at 19:11
  • 1
    $\begingroup$ Re the index 2 case: fix any symmetric generating set $T_0$ of $H$ such that $1 \in T_0$ and an element $g \in G \setminus H$. Take $$S = \{gt : t \in T_0\} \cup \{tg^{-1} : t \in T_0\}.$$ Then $S$ is a symmetric generating set of $G$ (we can get any $t \in T_0$ from a length-2 word in $S$, so we can generate $H$, and we get the coset $gH$) and an $S$-word represents an element of $H$ if and only if its length is even. $\endgroup$ Nov 19, 2014 at 20:00

1 Answer 1

2
$\begingroup$

(Edit:) This answers a different question, without the "finite index" assumption.

I think I can answer all three questions in the negative. Take for $G$ the Heisenberg group $$ G = \left(\begin{matrix} 1 & {\mathbb Z} & {\mathbb Z} \\ 0 & 1 & {\mathbb Z} \\ 0 & 0 & 1\end{matrix}\right) $$ and for $H$ its center (which is the $\mathbb Z$ sitting in the upper right corner). Then for any symmetric generating set $S$ of $G$ and any $z \in {\mathbb Z} \cong H$, we have $|z|_S \ll \sqrt{|z|}$, whereas for any symmetric generating set $T$ of $H$, we have $|z|_T \gg |z|$. Note that both $G$ and $H$ are nilpotent.

$\endgroup$
4
  • $\begingroup$ Thanks, but I'm interested in the case when $H$ is of finite index inside $G$, as written in the beginning. $\endgroup$
    – Miel Sharf
    Nov 16, 2014 at 15:52
  • $\begingroup$ OK, sorry, I had overlooked this assumption. The counter-example to 1 is still vaild, though. $\endgroup$ Nov 16, 2014 at 15:54
  • $\begingroup$ Could you please elaborate more? $\endgroup$
    – Miel Sharf
    Nov 16, 2014 at 15:56
  • $\begingroup$ The counter-example is in the comment to yor question. Can you be more precise as to what you would like to see more details on? $\endgroup$ Nov 16, 2014 at 15:59

Your Answer

By clicking “Post Your Answer”, you agree to our terms of service and acknowledge you have read our privacy policy.

Not the answer you're looking for? Browse other questions tagged or ask your own question.